11
$\begingroup$

I'm studying Riemannian manifolds that admit an almost-complex structure, thus $$3\tau+2\chi=c_1^2,$$ where $\tau$ is the signature, $\chi$ is the euler characteristic and $c_1$ is the first Chern class.

I know from Hizerbruch theorem that $$3\tau=p_1,$$ where $p_1$ is the first Pontryagin class. Notice that $\chi=c_2$, thus $$p_1=c_1^2-2c_2.$$ But I know from Chern-Weil Theory that $$c_1=\frac{iTr(F_A)}{2\pi},$$

$$c_2=\frac{Tr(F_A^2)-Tr(F_A)^2}{8\pi^2}$$ and $$p_1=-\frac{Tr(F_A^2)}{8\pi^2}$$. Here, $F_A^2=F_A\wedge F_A$ and $c_1^2=c_1\wedge c_1$. Computing, we have: $$c_1^2-2c_2=-\frac{Tr(F_A)^2}{4\pi^2}+\frac{Tr(F_A)^2-Tr(F_A^2)}{4\pi^2}=-\frac{Tr(F_A^2)}{4\pi^2}=2p_1.$$ What did I miss?

$\endgroup$
2
  • 2
    $\begingroup$ Hmm, a bonus factor of 2. Easy to lose/gain those when dealing with normalisations. $\endgroup$
    – David Roberts
    May 21, 2014 at 2:01
  • 1
    $\begingroup$ I want understanding why this factor appears. $\endgroup$
    – Geom math
    May 21, 2014 at 2:06

1 Answer 1

22
$\begingroup$

It follows from the definition $p_k(E) := (-1)^kc_{2k}(E \otimes \mathbb{C})$ that the formula $$p_1 = c_1^2 - 2c_2$$ is valid for all complex vector bundles (rather than just the tangent bundle of an almost complex surface).

The problem in your Chern-Weil calculation is essentailly that you are confusing the trace of a complex and a real matrix. A complex $n \times n$ matrix $M$ can be turned into a real $2n \times 2n$ matrix $M_\mathbb{R}$, and the traces are related by $$ {\rm Tr} \, M_\mathbb{R} = 2 \, {\rm Re} \, {\rm Tr} \, M . $$ If you take a complex connection $A$ on a complex vector bundle $E$, and let $A_\mathbb{R}$ be the induced connection on the underlying real vector bundle $E_\mathbb{R}$, then the correct interpretation of the Chern-Weil formula is that $p_1(E_\mathbb{R})$ is represented by $$ -\frac{{\rm Tr}(F_{A_\mathbb{R}}^2)}{8\pi^2} = -{\rm Re}\frac{{\rm Tr}(F_A^2)}{4\pi^2} . $$ Since the cohomology class ${\rm Tr}(F_A^2)$ is real that corrects the final step in your calculation.

$\endgroup$
0

Your Answer

By clicking “Post Your Answer”, you agree to our terms of service and acknowledge you have read our privacy policy.

Not the answer you're looking for? Browse other questions tagged or ask your own question.